Author Topic: EM Drive Developments - related to space flight applications - Thread 5  (Read 1313743 times)

Offline Notsosureofit

  • Full Member
  • ****
  • Posts: 691
  • Liked: 747
  • Likes Given: 1729
Notsosureofit:

The integrated copper frustum test article's -3dB loaded Q-factor for the 80W / ~100uN test runs or 1.25 uN/W was 7,100.  That is 1.25 uN/W / 3.33 nano-Newton (nN) / Watt = ~375.4 times as much thrust as a 100% efficient E&M rocket can produce.


Thanks Paul,  plugging in those numbers I get an unloaded Q of 10,900.

Offline Star-Drive

  • Member
  • Full Member
  • ****
  • Posts: 925
  • TX/USA
  • Liked: 1031
  • Likes Given: 31
Paul March,

Some people on this thread have been having trouble accepting that the emDrive requires new physics to explain its thrust if it is not experimental error. Could you please clarify for them that explanations such as this page are accurate descriptions of electrodynamics, and something else (quantum vacuum, gravitational warping, dark matter, or other effects not recorded to date) would be required to explain any anomylous thrust? I think this would help discussion on the emDrive to be much more productive.

Paul said in previous threads that Dr White, lead Eagleworks scientist, favors the Quantum Vacuum Fluctuation conjecture to explain the anomalous EmDrive thrust, implying virtual particles could be pushed on with Lorentz forces like a virtual magnetohydrodynamic drive (this is "new physics" in the sense that the accepted physics currently views the vacuum state as being immutable).

Also, Paul himself said several times here that he thinks the EmDrive thrust could originate from a Mach effect. I let you judge if this should be considered as "new physics" or more like some "upgraded old physics" as Mach's principle can be integrated into the "plain vanilla" general theory of relativity without referring to any quantum field like QVF. However, GRT with Mach's principle indeed requires an "action at a distance" field, a flavor of advanced/retarded waves of the Wheeler–Feynman absorber theory.

Personally I like this M-E theory more than QVF since it is CoM-savvy, as momentum transferred to the drive here is exchanged with the chiefly distant matter there (in the rest of the universe, through the gravinertial advanced/retarded Wheeler-Feynman field): CoM is preserved. Whereas with QFV, CoM seems broken since you are pushing off virtual particles that completely disappear after they have been pushed on.


Any thrust greater than 3.33 nano-Newtons per Watt would require new physics. I'm always hoping for new physics since it could revolutionize space travel, but unfortunately solid new physics results are pretty rare.



Glad to see we agree in principle.  Where is your calculation of  3.33 nano-Newtons per Watt ?

That is just the efficiency of a photon rocket. I don't feel like typing Greek letters, so the derivation is here. (Note that calculation is for a mirror, so it has double the result.)
Paul March,

Some people on this thread have been having trouble accepting that the emDrive requires new physics to explain its thrust if it is not experimental error. Could you please clarify for them that explanations such as this page are accurate descriptions of electrodynamics, and something else (quantum vacuum, gravitational warping, dark matter, or other effects not recorded to date) would be required to explain any anomylous thrust? I think this would help discussion on the emDrive to be much more productive.

Paul said in previous threads that Dr White, lead Eagleworks scientist, favors the Quantum Vacuum Fluctuation conjecture to explain the anomalous EmDrive thrust, implying virtual particles could be pushed on with Lorentz forces like a virtual magnetohydrodynamic drive (this is "new physics" in the sense that the accepted physics currently views the vacuum state as being immutable).

Also, Paul himself said several times here that he thinks the EmDrive thrust could originate from a Mach effect. I let you judge if this should be considered as "new physics" or more like some "upgraded old physics" as Mach's principle can be integrated into the "plain vanilla" general theory of relativity without referring to any quantum field like QVF. However, GRT with Mach's principle indeed requires an "action at a distance" field, a flavor of advanced/retarded waves of the Wheeler–Feynman absorber theory.

Personally I like this M-E theory more than QVF since it is CoM-savvy, as momentum transferred to the drive here is exchanged with the chiefly distant matter there (in the rest of the universe, through the gravinertial advanced/retarded Wheeler-Feynman field): CoM is preserved. Whereas with QFV, CoM seems broken since you are pushing off virtual particles that completely disappear after they have been pushed on.

Flux Capacitor:

I have tried to stay neutral in my support of either Woodward's M-E or White's Quantum Vacuum (QV) conjectures in regards to how CoM is being saluted in these frustum space drives.  I will point out to you though about something you said that the virtual particles not being able to convey momentum, could be in error.  It's true that a single charged virtual particle, most likely an electron or positron due to their low mass, only have a very, very limited lifetime in our universe.  However IF they collide and annihilate with a different virtual charged particle than they were created with, then any E&M field induced accelerations while they ARE in this universe should be carried forward as newly added momentum in the created gamma photon that continues on the creation / annihilation process.  If repeated a sufficient number of times, a momentum wake could be established in the QV, just like the momentum wake created in water by a propeller.  One form of this QVF momentum transfer mechanism is discussed in a Rice University paper by Dr. Paul M. Stevenson, (The Hydrodynamics of the Vacuum), that I pointed to before, see attached.  One could also view this QV momentum transfer mechanism as a higher dimensional interaction such as discussed in Dr. White's Physics Essays paper found here:

http://physicsessays.org/browse-journal-2/product/1396-11-harold-sonny-white-a-discussion-on-characteristics-of-the-quantum-vacuum.html   

Or the EW group paper found here:

http://ntrs.nasa.gov/archive/nasa/casi.ntrs.nasa.gov/20150006842.pdf

However all these conjectures require the QV to be mutable and degradable and/or we live in a 5, 6 or even more dimensional universe, which includes Woodward's reliance on Wheeler/Feynman radiations reaction forces, so your mileage may vary... 

Best, Paul March
« Last Edit: 11/01/2015 01:09 pm by Star-Drive »
Star-Drive

Offline X_RaY

  • Full Member
  • ****
  • Posts: 852
  • Germany
  • Liked: 1146
  • Likes Given: 2479
Paul March,

Some people on this thread have been having trouble accepting that the emDrive requires new physics to explain its thrust if it is not experimental error. Could you please clarify for them that explanations such as this page are accurate descriptions of electrodynamics, and something else (quantum vacuum, gravitational warping, dark matter, or other effects not recorded to date) would be required to explain any anomylous thrust? I think this would help discussion on the emDrive to be much more productive.

Paul said in previous threads that Dr White, lead Eagleworks scientist, favors the Quantum Vacuum Fluctuation conjecture to explain the anomalous EmDrive thrust, implying virtual particles could be pushed on with Lorentz forces like a virtual magnetohydrodynamic drive (this is "new physics" in the sense that the accepted physics currently views the vacuum state as being immutable).

Also, Paul himself said several times here that he thinks the EmDrive thrust could originate from a Mach effect. I let you judge if this should be considered as "new physics" or more like some "upgraded old physics" as Mach's principle can be integrated into the "plain vanilla" general theory of relativity without referring to any quantum field like QVF. However, GRT with Mach's principle indeed requires an "action at a distance" field, a flavor of advanced/retarded waves of the Wheeler–Feynman absorber theory.

Personally I like this M-E theory more than QVF since it is CoM-savvy, as momentum transferred to the drive here is exchanged with the chiefly distant matter there (in the rest of the universe, through the gravinertial advanced/retarded Wheeler-Feynman field): CoM is preserved. Whereas with QFV, CoM seems broken since you are pushing off virtual particles that completely disappear after they have been pushed on.


Any thrust greater than 3.33 nano-Newtons per Watt would require new physics. I'm always hoping for new physics since it could revolutionize space travel, but unfortunately solid new physics results are pretty rare.



Glad to see we agree in principle.  Where is your calculation of  3.33 nano-Newtons per Watt ?

That is just the efficiency of a photon rocket. I don't feel like typing Greek letters, so the derivation is here. (Note that calculation is for a mirror, so it has double the result.)
Paul March,

Some people on this thread have been having trouble accepting that the emDrive requires new physics to explain its thrust if it is not experimental error. Could you please clarify for them that explanations such as this page are accurate descriptions of electrodynamics, and something else (quantum vacuum, gravitational warping, dark matter, or other effects not recorded to date) would be required to explain any anomylous thrust? I think this would help discussion on the emDrive to be much more productive.

Paul said in previous threads that Dr White, lead Eagleworks scientist, favors the Quantum Vacuum Fluctuation conjecture to explain the anomalous EmDrive thrust, implying virtual particles could be pushed on with Lorentz forces like a virtual magnetohydrodynamic drive (this is "new physics" in the sense that the accepted physics currently views the vacuum state as being immutable).

Also, Paul himself said several times here that he thinks the EmDrive thrust could originate from a Mach effect. I let you judge if this should be considered as "new physics" or more like some "upgraded old physics" as Mach's principle can be integrated into the "plain vanilla" general theory of relativity without referring to any quantum field like QVF. However, GRT with Mach's principle indeed requires an "action at a distance" field, a flavor of advanced/retarded waves of the Wheeler–Feynman absorber theory.

Personally I like this M-E theory more than QVF since it is CoM-savvy, as momentum transferred to the drive here is exchanged with the chiefly distant matter there (in the rest of the universe, through the gravinertial advanced/retarded Wheeler-Feynman field): CoM is preserved. Whereas with QFV, CoM seems broken since you are pushing off virtual particles that completely disappear after they have been pushed on.

Flux Capacitor:

I have tried to stay neutral in my support of either Woodward's M-E or White's Quantum Vacuum (QV) conjectures in regards to how CoM is being saluted in these frustum space drives.  I will point out to you though about something you said that the virtual particles not being able to convey momentum, could be in error.  It's true that the a single charged virtual particle, most likely an electron or positron due to their low mass, only have a very, very limited lifetime in our universe.  However IF they collide and annihilate with a different virtual charged particle than they were created with, then any E&M field induced accelerations while they ARE in this universe should be carried forward as newly added momentum in the created gamma photon that continues on the creation / annihilation process.  If repeated a sufficient number of times, a momentum wake could be established in the QV, just like the momentum wake created in water by a propeller.  One form of this QVF momentum transfer mechanism is discussed in a Rice University paper by Dr. Paul M. Stevenson, (The Hydrodynamics of the Vacuum), that I pointed to before, see attached.  One could also view this QV momentum transfer mechanism as a higher dimensional interaction such as discussed in Dr. White's Physics Essays paper found here:

http://physicsessays.org/browse-journal-2/product/1396-11-harold-sonny-white-a-discussion-on-characteristics-of-the-quantum-vacuum.html   

Or the EW group paper found here:

http://ntrs.nasa.gov/archive/nasa/casi.ntrs.nasa.gov/20150006842.pdf

However all these conjectures require the QV to be mutable and degradable and/or we live in a 5, 6 or even more dimensional universe, which includes Woodward's reliance on Wheeler/Feynman radiations reaction forces, so your mileage may vary... 

Best, Paul March
At this hf frequencyies the preferred direction of the external EM field changes billions of times every second.
The average force acting on actual (virtual/real) charged particles have to be zero. At time t=0 the field pushes against that particle in a specific direction and after a half wavelength at t=1 the field and therefore the force turns by 180°, while the number of virtual particles remains over the time / is approximately constant.
And the creation of this particles is symmetrically (electron-<> positron+) the field acts on both in different directions, alone on this fact the average force have to be zero.

If I'm complete wrong please let me know. Maybe I miss some other specific reasonable facts. ???
« Last Edit: 10/31/2015 08:48 pm by X_RaY »

Offline flux_capacitor

  • Full Member
  • ****
  • Posts: 708
  • France
  • Liked: 860
  • Likes Given: 1076
{snip}
At this hf frequencyies the preferred direction of the external EM field changes billions of times every second.
The average force acting on actual (virtual/real) charged particles have to be zero. At time t=0 the field pushes against that particle in a specific direction and after a half wavelength at t=1 the field and therefore the force turns by 180°, while the number of virtual particles remains over the time / is approximately constant.
And the creation of this particles is symmetrically (electron-<> positron+) the field acts on both in different directions, alone on this fact the average force have to be zero.

If I'm complete wrong please let me know.

If you do reverse the electrical field E AS WELL AS the magnetic field B at the same time, then their cross-product the Lorentz Force F is always pointing in the same direction, and accelerate in the same direction both positively charged and negatively charged particles q moving through those fields at a velocity v:

F = q [ E + (v × B) ]

That's the principle of a magnetohydrodynamic drive (liquid metal, salt water or plasma-based) which can work with steady-state or pulsed DC fields, as well as AC fields.

But let's just use the more convenient Fleming's left-hand rule for motors, replacing the first part of the equation with the equivalent electrical current flow I:





I imagine the electrical and magnetic components of the EM waves in the resonant cavity, and the resulting Poynting vector, act the same way?
« Last Edit: 10/31/2015 09:03 pm by flux_capacitor »

Offline X_RaY

  • Full Member
  • ****
  • Posts: 852
  • Germany
  • Liked: 1146
  • Likes Given: 2479
{snip}
At this hf frequencyies the preferred direction of the external EM field changes billions of times every second.
The average force acting on actual (virtual/real) charged particles have to be zero. At time t=0 the field pushes against that particle in a specific direction and after a half wavelength at t=1 the field and therefore the force turns by 180°, while the number of virtual particles remains over the time / is approximately constant.
And the creation of this particles is symmetrically (electron-<> positron+) the field acts on both in different directions, alone on this fact the average force have to be zero.

If I'm complete wrong please let me know.

If you do reverse the electrical field E AS WELL AS the magnetic field B at the same time, then their cross-product the Lorentz Force F is always pointing in the same direction, and accelerate in the same direction both positively charged and negatively charged particles q moving through those fields at a velocity v:

F = q [ E + (v × B) ]

That's the principle of a magnetohydrodynamic drive (liquid metal, salt water or plasma-based) which can work with steady-state or pulsed DC fields, as well as AC fields.

I imagine the electrical and magnetic components of the EM waves in the resonant cavity, and the resulting Poynting vector, act the same way?
The field fluctuates over the time(E>-E; B>-B) and  q is minus for electrons and plus for positrons, right?
https://de.wikipedia.org/wiki/Lorentzkraft ; https://en.wikipedia.org/wiki/Lorentz_force

Translation of wikipedia(DE)

pic 1 b)
"... When deflection of a particle of charge q in spatially and temporally constant magnetic field as opposed to the deflection in the electric field no work is done, the kinetic energy and therefore the web speed so remain unchanged ..."

pic 1 a) and pic 2
"...Since the direction of the Lorentz force depends on the sign of the charge q, oppositely charged point charges the same direction of movement are deflected in opposite directions. ..."
« Last Edit: 10/31/2015 09:24 pm by X_RaY »

Offline meberbs

  • Senior Member
  • *****
  • Posts: 3096
  • Liked: 3379
  • Likes Given: 777
...

 Meberbs:

"Some people on this thread have been having trouble accepting that the emDrive requires new physics to explain its thrust if it is not experimental error."

I concur with your position that Maxwell's Classical E&M can NOT explain the frustum test results we continue to see, because when you sum up ALL of the Maxwell pressure tensors in the frustum due to all the E&M fields bouncing around inside the cavity and their interactions with any interior components like the PE discs and the active copper layer in the frustum's end and side walls, the NET force answer has to be ZERO by definition.  In other words classical E&M cannot provide an explanation for conservation of momentum for a closed E&M system that produces a net thrust.
If anyone feels like beating their head on the desk and run through this chapter to prove that Paul is right, be my guest. To save you knots, I'll give you a clue it's not there. https://archive.org/stream/ClassicalElectrodynamics/Jackson-ClassicalElectrodynamics#page/n253/mode/2up

So, there seems to be a hole, for something we don't quite understand...yet.

If you want to work through the proof you should start here. Sections 6.8 and 6.9 cover how to calculate momentum and forces in the presence of electrodynamic fields.

You can then integrate the forces on each wall of the cavity (applying spherical coordinates and using the techniques in chapter 8 like Greg Egan did to determine the fields), you can also save some work by noting that the fields will be sinusoidal in time for any resonating shape, so the momentum transfer to the walls will always average out to 0. Alternatively you can just note the entire theory is grounded in conservation of momentum since CoM is used in the derivation for the momentum stored in the fields.

Offline flux_capacitor

  • Full Member
  • ****
  • Posts: 708
  • France
  • Liked: 860
  • Likes Given: 1076
{snip}
At this hf frequencyies the preferred direction of the external EM field changes billions of times every second.
The average force acting on actual (virtual/real) charged particles have to be zero. At time t=0 the field pushes against that particle in a specific direction and after a half wavelength at t=1 the field and therefore the force turns by 180°, while the number of virtual particles remains over the time / is approximately constant.
And the creation of this particles is symmetrically (electron-<> positron+) the field acts on both in different directions, alone on this fact the average force have to be zero.

If I'm complete wrong please let me know.

If you do reverse the electrical field E AS WELL AS the magnetic field B at the same time, then their cross-product the Lorentz Force F is always pointing in the same direction, and accelerate in the same direction both positively charged and negatively charged particles q moving through those fields at a velocity v:

F = q [ E + (v × B) ]

That's the principle of a magnetohydrodynamic drive (liquid metal, salt water or plasma-based) which can work with steady-state or pulsed DC fields, as well as AC fields.

I imagine the electrical and magnetic components of the EM waves in the resonant cavity, and the resulting Poynting vector, act the same way?
The field fluctuates over the time(E>-E; B>-B) and  q is minus for electrons and plus for positrons, right?
https://de.wikipedia.org/wiki/Lorentzkraft ; https://en.wikipedia.org/wiki/Lorentz_force

Translation of wikipedia(DE)

pic 1 b)
"... When deflection of a particle of charge q in spatially and temporally constant magnetic field as opposed to the deflection in the electric field no work is done, the kinetic energy and therefore the web speed so remain unchanged ..."

pic 1 a) and pic 2
"...Since the direction of the Lorentz force depends on the sign of the charge q, oppositely charged point charges the same direction of movement are deflected in opposite directions. ..."

Yes, all you've said is true, but you're describing a magnetohydrodynamic generator, not a motor.

Imagine a plasma flowing into a duct. Take two opposite charges in that flow, moving together in the same direction, with the flow, and entering a zone where a magnetic field is applied. Both charges will feel a Lorentz force in the opposite direction, and those forces will separate the charges like in your pictures. They can then be collected though electrodes on the sides of the duct. This is a power source, an MHD generator:




Now, imagine a ionized gas at rest. Take the same duct and applied magnetic field, but this time, apply an electric field between the electrodes on the sides of the duct. unlike the first example, your opposite charges will flow in opposite directions: the electron will be accelerated towards the positive electrode, while the positron will be accelerated towards the negative electrode, because this time there is an applied E-field. The Lorentz force will accelerate both particles in the same direction. This is an electrical motor, an MHD drive:



The difference is when you apply or not an electric field. In an MHD drive, the electric field accelerates the charges in opposite directions, but the magnetic field curves their trajectory and the resulting Lorentz force pushes all charges, whatever their sign, in the same direction. And, through collisions in the gas, momentum is transferred to heavy ions and neutral atoms which are also accelerated (as a side note, magnetohydrodynamic thrusters are way more powerful than ion thrusters, the latter accelerating only one species, positive ions, through an electric field).

Basically it's the same thing as in a classical electric generator or linear motor with copper rotor and stator. Except with MHD your rotor is a working fluid (electrically conductive liquid or gas).
« Last Edit: 10/31/2015 10:52 pm by flux_capacitor »

Offline TheTraveller

Vax,

Have done a prelim search of available X band Rf amp chips. Found a few nice chips to use. Suggest the simplest way to use your high short term power delivery system and achieve a good dynamic effective thrust range is to be able to use pulse width and duty cycle modulation of the Rf stream feeding the frustum.

Should be able to achieve averaged thrust levels from sub mN to around 25mN this way.

Should also be able to build this into a rotary test rig that I can fit into a small 1 torr vac chamber. Is this vac low enough for qualification testing or do you require a lower vac?

We also need to establish a command and control protocol plus lock down a cubesat bus structure so what I build, you can control.

Have done cubesat work for my son's uni cubesat project so I have a basic understanding of the bus, well that was 2 years ago. Will spend the rest of the weekend getting up to speed on the latest cubesat standards.

I'm very confident this is doable:

1) totally fits into a standard 1U cubesat frame.
2) single pcb plugs into, powered and controlled by cubesat pcb connector.
3) uses pulse width and duty cycle modulation to achieve wide range of effective thrust upto around 25mN from integrated 60w X band Rf amp subsystem.
4) frustum resonance tracking via real time tracking of lowest VSWR.
5) will be qualified in a rotary 1 torr vac chamber test rig.
6) total mass under 1kg.
7) 1st rotary test before end 1st qtr 2016.

Comments
« Last Edit: 10/31/2015 10:50 pm by TheTraveller »
It Is Time For The EmDrive To Come Out Of The Shadows

Offline TheTraveller

Vax,

Your 10mN at 35W max input power requirements appears to be doable. Going to pulsed op at upto 50mN seems doable. 3kg is heaps of mass budget. All the electronics would be on one cubesat pcb with the frustum mounted and secured to the 1u modules frame. What g and vibration freq rates will the thruster and mounting system need to be designed to handle?

What are the processes to move forward, what are the precursor qualification requirements and what are the time frames as an overview?

Yes of course I need to do the rotary demo rig. That is a unspoken given requirement. Despite others opinion here, the EMDrive does work and this cubesat thruster is really doable.

It is my intention to start commercial sales of EMDrives, so the cubesat project will be done with commercial sales as the objective. It will be a high quality and high fidelity build.

Traveler:

We looked at using a 3U CubeSat as a means of validating the EmDrive physics, but the cost just for the required parts to build it is still well beyond our current means, even considering that the EW Lab could get a semi-free ride into orbit on one of the ISS resupply runs.  (The ISS can and does launch 3U CubeSats from the ISS Japanese lab module.)  Since you are considering selling CubeSats commercially, have you priced out how much a 3U at 3kg, 6U at 6kg and 12U at 12kg CubeSat would cost to have it put into orbit even using secondary payload status on flights of opportunity? 

I'm curious...

Best, Paul March

Hi Paul,

I'm very confident this is doable:

1) is supplied as a bolt on 1U cubesat propulsion module, with top end thrust, depending on pwr availibility, of around 25mN.

2) single pcb plugs into, powered and controlled by cubesat pcb connector.

3) uses pulse width and duty cycle modulation to achieve wide range of effective thrust upto around 25mN from an integrated 60w X band Rf amp subsystem that has self protection from thermal over temp.

4) real time frustum resonance tracking via lowest VSWR.

5) each thruster unit will be qualified on a rotary test rig, in a min 1 torr vac chamber.

My business model is to build and supply these 1U form factor X band thrusters to clients who wish to fly them as bolt on propulsion modules for their cubesat projects.

It is good to see you active on the forum again.


Phil Wilson
« Last Edit: 10/31/2015 11:33 pm by TheTraveller »
It Is Time For The EmDrive To Come Out Of The Shadows

Offline TheTraveller

Any thrust greater than 3.33 nano-Newtons per Watt would require new physics.

That opinion is not shared by either Roger Shawyer nor Prof Yang. Both of who have a very considerable depth of experimental experience and data. Probably exceeding that of Eagleworks.

Just maybe you need to review ALL their experimental data and theories that both claim to show no new physics is needed to explain the "Shawyer Effect".
It Is Time For The EmDrive To Come Out Of The Shadows

Offline SeeShells

  • Senior Member
  • *****
  • Posts: 2442
  • Every action there's a reaction we try to grasp.
  • United States
  • Liked: 3186
  • Likes Given: 2708
...

 Meberbs:

"Some people on this thread have been having trouble accepting that the emDrive requires new physics to explain its thrust if it is not experimental error."

I concur with your position that Maxwell's Classical E&M can NOT explain the frustum test results we continue to see, because when you sum up ALL of the Maxwell pressure tensors in the frustum due to all the E&M fields bouncing around inside the cavity and their interactions with any interior components like the PE discs and the active copper layer in the frustum's end and side walls, the NET force answer has to be ZERO by definition.  In other words classical E&M cannot provide an explanation for conservation of momentum for a closed E&M system that produces a net thrust.
If anyone feels like beating their head on the desk and run through this chapter to prove that Paul is right, be my guest. To save you knots, I'll give you a clue it's not there. https://archive.org/stream/ClassicalElectrodynamics/Jackson-ClassicalElectrodynamics#page/n253/mode/2up

So, there seems to be a hole, for something we don't quite understand...yet.

If you want to work through the proof you should start here. Sections 6.8 and 6.9 cover how to calculate momentum and forces in the presence of electrodynamic fields.

You can then integrate the forces on each wall of the cavity (applying spherical coordinates and using the techniques in chapter 8 like Greg Egan did to determine the fields), you can also save some work by noting that the fields will be sinusoidal in time for any resonating shape, so the momentum transfer to the walls will always average out to 0. Alternatively you can just note the entire theory is grounded in conservation of momentum since CoM is used in the derivation for the momentum stored in the fields.

Meberbs, I'm going to pass. If I felt a keener edge and could do it like I did 40 years ago I'd go at it. Just too darn rusty. You are right they are grounded in CoM.

Offline frobnicat

  • Full Member
  • ****
  • Posts: 518
  • Liked: 500
  • Likes Given: 151
.../...
Not being satisfied with just this analytical impulsive vs thermal signal separation approach, we are now working on a new integrated test article subsystem mounting arrangement with a new phase-change thermal management subsystem that should mitigate this thermally induced TP cg baseline shift problem once and for-all.

And yet the anomalous thrust signals remain...

Best, Paul March

Paul, thank you for posting news (within the constraints).

Following one of the aspect of the sensitivity of the experiment to cg shifts (discussed previously before the "black out") : do you still need a slight tilt (wrt strict verticality) of the axis of rotation of the TP's arm to have a stable rest position ? What else could be the cause for this thermally induced TP cg baseline shift problem that seems to indicate a measurement coupling directly proportional to temperature ?

The dynamic recoil of a part's mass accelerating (relative to fixation on TP's arm) under effect of thermal dilatation could hardly "shift" rest position at levels of 100µN for more than a few seconds. For instance a 1kg part would have to accelerate at 10-4 m/s˛, after 10s this would be a huge 5mm move. I am under the impression that a horizontal pendulum with strict verticality of axis of rotation (i.e. where there is no change in altitude of the test article when the arm rotates) should be sensitive only to such "dynamic recoils" effects, relatively easy to tame and filter out because proportional to second derivative (wrt time) of temperature.

Also, rules permitting, can you tell us if you have done new tests without dielectric inserts, and if yes if those tests are still yielding no measurable effect as for the previous experimental campaign at EW (Brady report 2014 and later follow-up by you on this forum) ?

Offline tleach

  • Member
  • Posts: 78
  • Berthoud, CO
  • Liked: 75
  • Likes Given: 105
Wow! Exciting times! TheTraveller is back. Paul March is back. Hooray!

I realize of course that I'm not in the same league as any of you, but here are some pictures of my DIY build progress. 5W at between 5.7-5.9 GHz, self contained and operated via a remote switch. I probably won't be able to get resonance until I shorten it quite a bit.

Right now the Riker Drive's small diameter is about 1.1 inches, large diameter is about 7.5 inches, and length is about 10.5 inches. These measurements will change as I progress through my testing, but can't start that until I build a test stand.

T. Thor Leach

Offline TheTraveller

129,000 euros for a 2 axis 1U cubesat cold gas thruster. AND you still need to buy the N2 cold gas canisters!

For sure can beat that price.
It Is Time For The EmDrive To Come Out Of The Shadows

Offline A_M_Swallow

  • Elite Veteran
  • Senior Member
  • *****
  • Posts: 8906
  • South coast of England
  • Liked: 500
  • Likes Given: 223
{snip}
I'm very confident this is doable:

1) totally fits into a standard 1U cubesat frame.
2) single pcb plugs into, powered and controlled by cubesat pcb connector.
3) uses pulse width and duty cycle modulation to achieve wide range of effective thrust upto around 25mN from integrated 60w X band Rf amp subsystem.
4) frustum resonance tracking via real time tracking of lowest VSWR.
5) will be qualified in a rotary 1 torr vac chamber test rig.
6) total mass under 1kg.
7) 1st rotary test before end 1st qtr 2016.

Comments

A second market for EM Drives is RCS for larger satellites. 25mN is low but gives high accuracy. Alternatively "burn" for an entire day.

v = u + a t so t = Δv / a
F = m a so a = F/m
combining t = Δv * m / F

To give a 1 tonne (1000 kg) satellite a delta-v change of 1 m/s
t = 1 * 1000 / 0.025 = 40,000 seconds (or 11.11 hours)

Offline TheTraveller

{snip}
I'm very confident this is doable:

1) totally fits into a standard 1U cubesat frame.
2) single pcb plugs into, powered and controlled by cubesat pcb connector.
3) uses pulse width and duty cycle modulation to achieve wide range of effective thrust upto around 25mN from integrated 60w X band Rf amp subsystem.
4) frustum resonance tracking via real time tracking of lowest VSWR.
5) will be qualified in a rotary 1 torr vac chamber test rig.
6) total mass under 1kg.
7) 1st rotary test before end 1st qtr 2016.

Comments

A second market for EM Drives is RCS for larger satellites. 25mN is low but gives high accuracy. Alternatively "burn" for an entire day.

v = u + a t so t = Δv / a
F = m a so a = F/m
combining t = Δv * m / F

To give a 1 tonne (1000 kg) satellite a delta-v change of 1 m/s
t = 1 * 1000 / 0.025 = 40,000 seconds (or 11.11 hours)

25mN is only limited by Rf amp watts and power supply availability.

Can always end stack the standard 25mN 1U modules for higher acceleration rates & built 3 axis arrays for attitude control.

Like space lego. Only they make the sat move.
It Is Time For The EmDrive To Come Out Of The Shadows

Offline LasJayhawk

I really really dislike the term "new physics". We aren't changing the workings of the universe, just getting an idea of something not yet understood.

Offline Fugudaddy

  • Member
  • Posts: 31
  • Rochester NY
  • Liked: 57
  • Likes Given: 70
And yet the anomalous thrust signals remain...
Best, Paul March

Reports of spooky movement, and on Halloween no less.

Thank you, Sir, for you and your team's work.

There will likely be debates for years about exactly what's causing this anomaly, but tests like these and papers like yours (and Shell and Dave and and and) only add to the reality that something may very well be happening that we can't (yet) explain by our currently understood science.

Floobie dust? Ghosts? Probably not. But there does seem to be something. Not knowing what or how it works doesn't mean it doesn't. :)

Offline Corlock Striker

  • Member
  • Posts: 13
  • New York, NY
  • Liked: 8
  • Likes Given: 1
However all these conjectures require the QV to be mutable and degradable and/or we live in a 5, 6 or even more dimensional universe, which includes Woodward's reliance on Wheeler/Feynman radiations reaction forces, so your mileage may vary... 

Best, Paul March

Paul,

You, Dr. Woodard, and Dr. White may find this article interesting.  It discusses a paper that is up for review on ArXiv that claims to have found evidence of our universe colliding with alternate universes in the Cosmic Microwave Background Radiation.  The actual paper on ArXiv is most likely beyond me, but I'm sure the three of you would be able to make sense of it.  I also imagine, you'd be able to tell if there was any truth to its claims.  Given your statements about the EmDrive possibly requiring us to live in a multi-dimensional universe, if this paper has actually found evidence of the existence of alternate universes, that may actually help to explain how the EmDrive functions.  Thought I'd make you aware of it, as it might be of help to you.

Offline TheTraveller

Interesting 1U CubeSat 10mN hot gas thruster form factor and performance profile:
http://www.busek.com/index_htm_files/70008518B.pdf

Now imagine a X band 10cm dia x 10cm long frustum installed in that 1U form factor.
« Last Edit: 11/01/2015 02:51 am by TheTraveller »
It Is Time For The EmDrive To Come Out Of The Shadows

Tags:
 

Advertisement NovaTech
Advertisement Northrop Grumman
Advertisement
Advertisement Margaritaville Beach Resort South Padre Island
Advertisement Brady Kenniston
Advertisement NextSpaceflight
Advertisement Nathan Barker Photography
1